www.vorhilfe.de
- Förderverein -
Der Förderverein.

Gemeinnütziger Verein zur Finanzierung des Projekts Vorhilfe.de.
Hallo Gast!einloggen | registrieren ]
Startseite · Mitglieder · Impressum
Forenbaum
^ Forenbaum
Status VH e.V.
  Status Vereinsforum

Gezeigt werden alle Foren bis zur Tiefe 2

Navigation
 Startseite...
 Suchen
 Impressum
Das Projekt
Server und Internetanbindung werden durch Spenden finanziert.
Organisiert wird das Projekt von unserem Koordinatorenteam.
Hunderte Mitglieder helfen ehrenamtlich in unseren moderierten Foren.
Anbieter der Seite ist der gemeinnützige Verein "Vorhilfe.de e.V.".
Partnerseiten
Weitere Fächer:

Open Source FunktionenplotterFunkyPlot: Kostenloser und quelloffener Funktionenplotter für Linux und andere Betriebssysteme
Forum "Mathe Klassen 8-10" - Logarithmengesetze
Logarithmengesetze < Klassen 8-10 < Schule < Mathe < Vorhilfe
Ansicht: [ geschachtelt ] | ^ Forum "Mathe Klassen 8-10"  | ^^ Alle Foren  | ^ Forenbaum  | Materialien

Logarithmengesetze: Erklärung
Status: (Frage) beantwortet Status 
Datum: 09:57 Mo 01.12.2014
Autor: marvinmarvin

Aufgabe
Vereinfache und berechne soweit wie möglich:

[mm] log_{a}\bruch{b}{c} [/mm] + [mm] log_{a}b [/mm]

Hallo zusammen,

ich lerne gerade für das Thema Logarithmen aber irgendwie habe ich das nicht verstanden.

In der Aufgabe oben habe ich gerechnet:

[mm] log_{a}(\bruch{b}{c} [/mm] * b)

und würde erhalten laut Logarithmengesetze:

[mm] log_{a}(\bruch{b^{2}}{c}) [/mm]

Aber laut Lösung ist das Ergebnis:

[mm] log_{a}(1+\bruch{b}{c}) [/mm]

Wieso?

Danke schon mal an alle!


Ich habe diese Frage in keinem Forum auf anderen Internetseiten gestellt.

        
Bezug
Logarithmengesetze: Antwort
Status: (Antwort) fertig Status 
Datum: 10:43 Mo 01.12.2014
Autor: angela.h.b.


> Vereinfache und berechne soweit wie möglich:

>

> [mm]log_{a}\bruch{b}{c}[/mm] + [mm]log_{a}b[/mm]
> Hallo zusammen,

>

> ich lerne gerade für das Thema Logarithmen aber irgendwie
> habe ich das nicht verstanden.

Hallo,

[willkommenmr].

>

> In der Aufgabe oben habe ich gerechnet:

[mm]log_{a}\bruch{b}{c}[/mm] + [mm]log_{a}b[/mm]=
>

> [mm]log_{a}(\bruch{b}{c}[/mm] * b)

>

> und würde erhalten laut Logarithmengesetze:

>

> [mm]log_{a}(\bruch{b^{2}}{c})[/mm]

Das ist auf jeden Fall richtig.
"Vereinfache" ist oftmals eine etwas schwammige Anweisung.
Mir ist nicht klar, ob Du den Ausdruck als nur einen Log schreiben sollst (dann bist Du fertig), oder als Summe möglichst einfacher Logarithmen.
Falls dies gefordert wäre, würde man noch so weitermachen:

[mm] ...=log_{a}(b^2)-log_a(c) [/mm]

[mm] =2*log_a(b)-log_a(c) [/mm]

>

> Aber laut Lösung ist das Ergebnis:

>

> [mm]log_{a}(1+\bruch{b}{c})[/mm]

Diese Lösung paßt nicht zu Deiner Aufgabe.
Sie ist falsch.
Sie paßt z.B. zur Aufgabe [mm] log_a(c+b)-log_a(c). [/mm]

LG Angela







Bezug
                
Bezug
Logarithmengesetze: Frage (beantwortet)
Status: (Frage) beantwortet Status 
Datum: 10:59 Mo 01.12.2014
Autor: marvinmarvin

Danke für die schnelle Antwort :-)

Dann haben die sich hier in der Lösung vertippt - auch ein Mathebuch kann sich irren :-)

Habe noch eine letzte Frage zu diesem Thema:

Wie wäre es bei so einer Aufgabe:

[mm] (log_{a} a^{2})^{-3} [/mm] + [mm] (log_{a} 1)^{3} [/mm]

mein Ansatz wäre erst Umschreiben in :

[mm] \bruch{1}{log_{a}^{3}a^{2}} [/mm] + [mm] log_{a}^{3}(1) [/mm]

dann würde ich alles auf einen Nenner bringen aber das bringt mir nichts weil ich dann auf der rechten Seite eine Multiplikation von log's hätte!

Also sowas hier:


[mm] \bruch{1}{log_{a}^{3}a^{2}} [/mm] + [mm] \bruch{log_{a}^{3}(1) *log_{a}^{3}a^{2} }{log_{a}^{3}a^{2}} [/mm]  


[mm] \bruch{log_{a}^{3}(1) *log_{a}^{3}a^{2} + 1 }{log_{a}^{3}a^{2}} [/mm]                              

Bezug
                        
Bezug
Logarithmengesetze: Antwort
Status: (Antwort) fertig Status 
Datum: 11:13 Mo 01.12.2014
Autor: chrisno

Achte auf die besonderen Werte: [mm] $log_{a}(a) [/mm] = ...$ und [mm] $log_{a}(1) [/mm] = ...$


Bezug
                                
Bezug
Logarithmengesetze: Frage (beantwortet)
Status: (Frage) beantwortet Status 
Datum: 12:14 Mo 01.12.2014
Autor: marvinmarvin

Also dann wäre folgendes:

Ausgangsgleichung:
[mm] (log_{a} a^{2})^{-3} [/mm] +  [mm] log_{a}^{3}(1) [/mm]

[mm] log_{a}^{3}(1) [/mm] = 0

[mm] \Rightarrow (log_{a} a^{2})^{-3} [/mm]

Umschreiben:
[mm] \Rightarrow \bruch{1}{log_{a}^{3}a^{2}} [/mm]

[mm] \Rightarrow \bruch{1}{2log_{a}^{3}a} [/mm]

[mm] log_{a}(a) [/mm] =1

aber weiter komme ich nicht die Lösung soll [mm] \bruch{1}{8} [/mm] sein.

:-(


Bezug
                                        
Bezug
Logarithmengesetze: Mitteilung
Status: (Mitteilung) Reaktion unnötig Status 
Datum: 12:16 Mo 01.12.2014
Autor: marvinmarvin

Ok, weiß nun wie :-)

Bezug
                                        
Bezug
Logarithmengesetze: Potenzgesetz beachten
Status: (Antwort) fertig Status 
Datum: 12:19 Mo 01.12.2014
Autor: Roadrunner

Hallo Marvin!


Du machst eine Klammerfehler: denn die Potenz [mm] $(...)^3$ [/mm] bezieht sich ja auf den gesamten Term (Stichwort: MBPotenzgesetze).

[mm] $\left[ \ \log_a\left(a^2\right) \ \right]^3 [/mm] \ = \ [mm] \left[ \ 2*\log_a(a) \ \right]^3 [/mm] \ = \ [mm] 2^3*\left[\log_a(a)\right]^3 [/mm] \ = \ ...$


Gruß vom
Roadrunner

Bezug
Ansicht: [ geschachtelt ] | ^ Forum "Mathe Klassen 8-10"  | ^^ Alle Foren  | ^ Forenbaum  | Materialien


^ Seitenanfang ^
ev.vorhilfe.de
[ Startseite | Mitglieder | Impressum ]